PT 27 LR 1 #7, 17, 19 Forum

Prepare for the LSAT or discuss it with others in this forum.
Post Reply
xqhp82

Bronze
Posts: 310
Joined: Wed Aug 19, 2009 12:37 pm

PT 27 LR 1 #7, 17, 19

Post by xqhp82 » Wed Aug 11, 2010 10:09 pm

I just did the first LR section in PT 27 and thought it was pretty manageable, until I found out that I scored a horrible -10....I'm rather confused about a few questions here.

#7 - can someone explain why it's C? I chose A because I thought reduced price-->more sales-->more magazine readers. but C doesn't show why people would go for less widely read magazines instead. am i missing something?

#17 - I understand how the claim is a premise and that supports the conclusion, but can it also be what (E) suggests, that it's an assumption of the argument?

#19 - I chose A- pollens and other allergens do not cause colds. i think i misinterpreted the last sentence in the stumuli. can someone clarify what that means exactly?

Thanks!

User avatar
matt@atlaslsat

New
Posts: 63
Joined: Wed Mar 31, 2010 2:34 pm

Re: PT 27 LR 1 #7, 17, 19

Post by matt@atlaslsat » Sat Aug 14, 2010 3:51 pm

Sorry for the lengthy responses. These answers are being written to be reposted on our forum as a part of our ever growing set of explanations to all LSAT questions.

Feel free to check us out at http://www.atlaslsat.com/forums for future questions!

PT27, S1, Q7 - Because of the recent recession

This question asks us to resolve a paradox. We're asked to explain how it could be that the most widely read magazines could be seeing such drastically different results than other magazines in Country A. Here are the two statements we're asked to reconcile.

1. the survival of the most widely read magazines is in grave doubt.
2. the number of financially successful magazines in Country A is greater than ever.

To do this, we'll need an answer choice that distinguishes the most widely read magazines from other magazines.

(A) affects both groups - the most widely read magazines and other magazines. Since it doesn't distinguish the most widely read magazines it can't explain the discrepant findings.
(B) is irrelevant. What concerns television shows plays no role in the apparent discrepancy from the stimulus.
(C) distinguishes the most widely read magazines from other magazines and provides a source for the discrepant findings - ad revenue vs. revenue from circulation.
(D) affects both groups - the most widely read magazines and other magazines. Since it doesn't distinguish the most widely read magazines it can't explain the discrepant findings.
(E) is irrelevant. Surviving does not equate to financially successful, so we don't have any points of comparison.

PT27, S1, Q17 - The stable functioning of

Great question! And the simple answer is no. A claim that is written explicitly can never be an assumption. An assumption is by definition unstated.

This question asks us to identify the role of the claim that a society is stable only if its laws tend to increase the happiness of its citizens.

The first sentence of this argument is the conclusion and can be identified by the words "this is clear from" that begin the second sentence. Everything after the first sentence is evidence for why the first sentence is true. Thus, answer choice (B) properly identifies the role of the claim.

(A) is not true, the first sentence is the conclusion.
(C) is not true, the claim does not challenge the conclusion, but rather supports it.
(D) could possibly be the correct answer to another argument, but the statement does not follow as a result of the conclusion, but rather supports the conclusion.
(E) would be accurate if the claim was an example of a general principle. In fact this claim is a general principle, not an illustration of one. We know this because of the words "only if."

PT27, S1, Q19 - Pollen and other allergens can

Tough question. And the fact that you were tempted by answer choice (A) does seem to indicate a misread of the last statement. We know that histamines play no role in the processes by which cold produce their symptoms, but that doesn't mean that histamines play no role in causing colds! A slight but important difference.

This question asks us to infer from the series of facts stated in the stimulus.

(A) cannot be inferred from the stimulus. We're given information as to what causes symptoms of colds, but not what causes colds themselves.
(B) is out of scope. The difficulty by which these conditions are treated is not related to us in the information.
(C) could be false. It could be the case that antihistamines are effective at treating congestion caused by colds.
(D) is a bit tempting because we know that antihistamines make you drowsy, but that's not the same thing as treating sleeplessness. Also, we don't even know that sleeplessness sometimes accompanies allergies!
(E) must be true. We can infer this from the last sentence. If histamines play no role in the processes by which colds produce their symptoms, then antihistamines that treat those symptoms would not be doing so by blocking the action of histamines.

xqhp82

Bronze
Posts: 310
Joined: Wed Aug 19, 2009 12:37 pm

Re: PT 27 LR 1 #7, 17, 19

Post by xqhp82 » Mon Aug 16, 2010 9:01 pm

thanks very much! they all look clear now.

Sandro

Gold
Posts: 2525
Joined: Sat Jul 18, 2009 12:12 am

Re: PT 27 LR 1 #7, 17, 19

Post by Sandro » Tue Aug 17, 2010 4:23 pm

Can anyone explain #25 from this section? Im having a hard time with it. Is B the right answer because pretending to have a hobby would fall under the argument that hobbies shouldnt be asked during interviews because they are unreliable as indicators?

User avatar
3|ink

Platinum
Posts: 7393
Joined: Wed Dec 16, 2009 5:23 pm

Re: PT 27 LR 1 #7, 17, 19

Post by 3|ink » Tue Aug 17, 2010 4:25 pm

Sandro777 wrote:Can anyone explain #25 from this section? Im having a hard time with it. Is B the right answer because pretending to have a hobby would fall under the argument that hobbies shouldnt be asked during interviews because they are unreliable as indicators?
I think it's the right answer because it's irrelevant.

Want to continue reading?

Register now to search topics and post comments!

Absolutely FREE!


User avatar
matt@atlaslsat

New
Posts: 63
Joined: Wed Mar 31, 2010 2:34 pm

Re: PT 27 LR 1 #7, 17, 19

Post by matt@atlaslsat » Tue Aug 17, 2010 5:14 pm

PT27, S1, Q25 - When interviewing job candidates

The core of this argument is that because enthusiasm for hobbies may indicate that the candidate is less concerned with work than with play, personnel managers should not inquire about a candidate's hobbies.

Just because such enthusiasm may have uncertain results regarding one aspect of a candidates job performance does not mean we can't use questions about hobbies to learn other important factors relating to job performance.

We're asked to find the answer choice that does not reflect a flaw in the argument.

(A) undermines the conclusion that personnel managers should not inquire inquire about hobbies.
(B) supports the conclusion that personnel managers should not inquire about hobbies. The reason being that personnel managers are not likely to get useful information, because they cannot count on getting honest answers.
(C) undermines the conclusion that personnel managers should not inquire inquire about hobbies.
(D) undermines the conclusion that personnel managers should not inquire inquire about hobbies.
(E) undermines the conclusion that personnel managers should not inquire inquire about hobbies.

Each of the incorrect answer choices offers a reason why personnel managers would want to inquire about hobbies, whereas the last actually works in the opposite direction by supporting the conclusion that they should not ask such questions. Does that help clear this one up?

Sandro

Gold
Posts: 2525
Joined: Sat Jul 18, 2009 12:12 am

Re: PT 27 LR 1 #7, 17, 19

Post by Sandro » Tue Aug 17, 2010 6:17 pm

Yes. Thank you.

Want to continue reading?

Register for access!

Did I mention it was FREE ?


Post Reply

Return to “LSAT Prep and Discussion Forum”